Vous êtes sur la page 1sur 14

Version 091 EX3 ditmire (58216) 1

This print-out should have 20 questions.


Multiple-choice questions may continue on
the next column or page nd all choices
before answering.
001 10.0 points
A uniform at plate of metal with mass 1 kg
is situated in the reference frame.
0 1 2 3 4 5 6 7 8 9 10
0
1
2
3
4
5
x ( m )
y
(
m
)
Calculate the moment of inertia of the tri-
angle with the y-coordinate as the axis of
rotation.
1. 8.0
2. 16.6667
3. 54.0
4. 30.0
5. 40.5
6. 24.0
7. 2.66667
8. 4.5
9. 12.5
10. 24.5
Correct answer: 2.66667 kg m
2
.
Explanation:
Let : (x
1
, y
1
) = (0 m, 0 m)
(x
2
, y
2
) = (4 m, 0 m)
(x
3
, y
3
) = (0 m, 3 m) .
The equation for the hypotenuse is
y y
2
= s (x x
2
)
=
y
3
x
2
(x x
2
) .
The moment of inertia of the triangle about
the x-coordinate is
I
x=0
y

_
x2
0
x
2
dm
=
_
x2
0
x
2
y dx
= s
_
x2
0
x
2
(x x
2
) dx
= s
_
1
4
x
4

1
3
x
2
x
3
_

x2
0
= s
_
1
4
x
4
2

1
3
x
4
2
_
=
_
2 m
x
2
y
3
_ _

y
3
x
2
_ _

1
12
x
4
2
_
=
1
6
mx
2
2
=
1
6
(1 kg) (4 m)
2
= 2.66667 kg m
2
.
Alternate Solution:
The x-coordinate of the center of mass is
x
cm
=

_
x2
0
x y dx

_
x2
0
y dx
=
_
x2
0
x s (x x
2
) dx
_
x2
0
s (x x
2
) dx
=
_
x2
0
x (x x
2
) dx
_
x2
0
(x x
2
) dx
=
1
3
x
3

1
2
(x
2
) x
2

x2
0
1
2
x
2
(x
2
) x

x2
0
=
1
3
x
3
2

1
2
(x
2
) x
2
2
1
2
x
2
2
(x
2
) x
2
=
x
3
2
3 x
2
2
=
1
3
x
2
.
The moment of inertia about the center of
mass is
I
x=xcm
y
=
_
x2
0
(x x
cm
)
2
dm
Version 091 EX3 ditmire (58216) 2
=
_
x2
0
(x x
cm
)
2
y dx
= s
_
x2
0
(x x
cm
)
2
(x x
2
) dx
= s
_
x2
0
_
x
3
(2 x
cm
+x
2
) x
2
+(x
2
cm
+ 2 x
cm
x
2
) x
x
2
cm
x
2
_
dx .
Since 2 x
cm
+x
2
=
2
3
x
2
+x
2
=
5
3
x
2
,
x
2
cm
+ 2 x
cm
x
2
=
1
9
x
2
2
+
2
3
x
2
2
=
7
9
x
2
2
,
and x
2
cm
x
2
=
1
9
x
3
2
, then
I
x=xcm
y
= s
_
x2
0
_
x
3

5
3
x
2
x
2
+
7
9
x
2
2
x

1
9
x
3
2
_
dx
= s
_
1
4
x
4

5
9
x
2
x
3
+
7
18
x
2
2
x
2

1
9
x
3
2
x
_

x2
0
=
_
2 m
x
2
y
3
_ _

y
3
x
2
_ _
1
4
x
4
2

5
9
x
4
2
+
7
18
x
4
2

1
9
x
4
2
_
=
2 m
x
2
2
_

1
36
x
4
2
_
=
1
18
mx
2
2
.
Using the parallel axis theorem,
I
x=0
y
= I
x=xcm
y
+mx
2
cm
=
1
18
mx
2
2
+m
_
1
9
x
2
2
_
=
1
6
mx
2
2
=
1
6
(1 kg) (4 m)
2
= 2.66667 kg m
2
.
002 10.0 points
A 13.4 g bullet is red into a 131 g wooden
block initially at rest on a horizontal surface.
The acceleration of gravity is 9.8 m/s
2
.
After impact, the block slides 4.77 m before
coming to rest.
131 g
13.4 g
v
bullet
144.4 g
4.77 m
= 0.697
If the coecient of friction between block
and surface is 0.697 , what was the speed of
the bullet immediately before impact?
1. 51.1939
2. 70.4463
3. 86.9893
4. 94.4178
5. 43.8826
6. 73.6531
7. 45.6235
8. 47.4633
9. 55.2294
10. 119.637
Correct answer: 86.9893 m/s.
Explanation:
Given : m
bullet
= 13.4 g ,
m
block
= 131 g ,
s = 4.77 m, and
= 0.697 .
Applying conservation of momentum to the
collision,
m
bullet
v = (m
bullet
+m
block
) v

Applying the work-kinetic energy theorem af-


ter the collision,
W
net
= W
fr
= KE
f
KE
i
f
k
s =
1
2
(m
bullet
+m
block
) v

k
(m
bullet
+m
block
) g s
=
1
2
(m
bullet
+m
block
) v

2
v

=
_
2
k
g s
Version 091 EX3 ditmire (58216) 3
Applying conservation of momentum to the
collision,
m
bullet
v = (m
bullet
+m
block
) v

v =
m
bullet
+m
block
m
bullet
_
2
k
g s
=
13.4 g + 131 g
13.4 g

_
2 (0.697) (9.8 m/s
2
) (4.77 m)
= 86.9893 m/s .
003 (part 1 of 2) 10.0 points
Given: A uniform exible chain whose mass
is 6.7 kg and length is 7 m. A table whose top
is frictionless.
Initially you are holding the chain at rest
and one-half of the length of the chain is hung
over the edge of the table. When you let loose
of the chain it falls downward.
The acceleration of gravity is 9.8 m/s
2
.
a
7
m
5
.
6
m
Radius of table
is negligible
compared to the
length of chain
Mass of chain
is 6.7 kg .
Find the acceleration a of the chain when
the length of the chain hanging vertically is
5.6 m.
1. 7.154
2. 7.742
3. 5.978
4. 6.468
5. 7.84
6. 6.664
7. 6.174
8. 7.056
9. 7.35
10. 7.252
Correct answer: 7.84 m/s
2
.
Explanation:
Note: The initial condition does not enter
into the consideration for the acceleration.
Let : g = 9.8 m/s
2
,
L = 7 m,
= 5.6 m, and
m = 6.7 kg .
The linear density of the chain is
=
m
L
=
6.7 kg
7 m
= 0.957143 kg/m.
F = g
cm
The free body diagram in the vertical di-
rection gives

F
y
= g = La .
Therefore
a =

L
g (1)
=
5.6 m
7 m
(9.8 m/s
2
)
= 7.84 m/s
2
.
004 (part 2 of 2) 10.0 points
Find the magnitude of the velocity of the of
Version 091 EX3 ditmire (58216) 4
the chain when 5.6 m of the chain is hanging
vertically.
1. 5.17243
2. 5.41567
3. 4.28661
4. 4.6901
5. 3.81458
6. 3.43999
7. 3.5
8. 3.72318
9. 4.18317
10. 2.57824
Correct answer: 5.17243 m/s.
Explanation:
The change in kinetic energy is
K =
1
2
mv
2
=
1
2
Lv
2
. (2)
Let
i
=
L
2
and
f
= .
Using the table top as the origin of the
y-coordinate and down as the positive y di-
rection
y
cm
=
m
on table
_
0
_
+m
hanging
_

2
_
m
on table
+m
hanging
y
cmi
=
_
L
L
2
_
0 +
L
2

_
L
4
_
L
y
cm
f
=
(L ) 0 +
_

2
_
L
The vertical center of mass dierence y
cm
is
y
cm
= y
cm
f
y
cmi
=


2

L
2
L
4
L
=
1
8 L
[4
2
L
2
] . (3)
The change in potential energy is
U = Lg y
cm
=
1
8
g [4
2
L
2
] . (4)
From conservation of energy K = U, Eq.
2 and Eq. 4, we have
1
2
Lv
2
=
1
8
g (4
2
L
2
)
v
2
=
g
4 L
[4
2
L
2
] . (5)
Therefore
v =
_
g
4 L
[4
2
L
2
] (6)
=

(9.8 m/s
2
)
4 (7 m)
[4 (5.6 m)
2
(7 m)
2
]
= 5.17243 m/s .
Alternative: You can use the kinematic
expression and remember that the center of
mass accelerates at g [not Eq. 1], since the
acceleration of gravity is not a function of
mass.
v
2
= 2 a y
cm
= 2 g
1
8 L
[4
2
L
2
]
=
g
4 L
[4
2
L
2
] , (7)
where y
cm
is obtained from Eq. 3.
Note: Equation 7 is identical to Eq. 5.
005 10.0 points
A 360 kg load is hung on a wire of length
4.9 m and cross sectional area 4 10
5
m
2
,
What is its increase in length? The ac-
celeration of gravity is 9.8 m/s
2
and Youngs
modulus is 6.6 10
10
N/m
2
.
1. 4.83878
2. 6.54818
3. 5.41288
4. 6.92868
5. 4.1295
6. 4.23457
7. 18.046
8. 7.14854
9. 5.66054
10. 4.31748
Correct answer: 6.54818 mm.
Version 091 EX3 ditmire (58216) 5
Explanation:
Let : m = 360 kg ,
g = 9.8 m/s
2
,
A = 4 10
5
m
2
,
= 4.9 m, and
Y = 6.6 10
10
N/m
2
.
F
A
= Y

=
mg
A

Y
=
(360 kg)(9.8 m/s
2
)
4 10
5
m
2
4.9 m
6.6 10
10
N/m
2

1000 mm
1 m
= 6.54818 mm .
006 10.0 points
A(n) 6 kg object moving with a speed of
8.3 m/s collides with a(n) 18 kg object moving
with a velocity of 10 m/s in a direction 23

from the initial direction of motion of the 6 kg


object.
8.3 m/s
1
0
m
/
s
18 kg
6 kg
23

What is the speed of the two objects after


the collision if they remain stuck together?
1. 7.05366
2. 8.73161
3. 8.61664
4. 7.70344
5. 10.4421
6. 9.73592
7. 9.05915
8. 9.44491
9. 9.98124
10. 10.2673
Correct answer: 9.44491 m/s.
Explanation:
Let : m
1
= 6 kg ,
m
2
= 18 kg ,
m
f
= m
1
+m
2
= 24 kg ,
v
1
= 8.3 m/s ,
v
2
= 10 m/s ,
p
1
= m
1
v
1
= 49.8 kg m/s ,
p
2
= m
2
v
2
= 180 kg m/s ,
p
1
p
2
= m
1
v
1
m
2
v
2
= 17928 kg
2
m
2
/s
2
,
p
x
= p
1
+p
2
cos
= 215.491 kg m/s ,
p
y
= p
2
sin
= 70.3316 kg m/s ,
= 23

, and
= 157

.
v
1
v2
m
2
m
1

vf

m
f
The nal momentum is
p
f
= (m
1
+m
2
) v
f
. (1)
Momentum is conserved
p
1
+p
2
=p
f
. (2)
Using the law of cosines, we have
p
2
1
+p
2
2
2 p
1
p
2
cos( ) = p
2
f
.
Solving for v
f
, we have
v
f
=
_
p
2
1
+p
2
2
2 p
1
p
2
cos( )
m
1
+m
2
=
_
1
(6 kg) + (18 kg)
_

_
(2480.04 kg
2
m
2
/s
2
)
2
+ (32400 kg
2
m
2
/s
2
)
2
(17928 kg
2
m
2
/s
2
) cos(157

)
_
1/2
= 9.44491 m/s .
Version 091 EX3 ditmire (58216) 6
Alternate Solution: Since
p
2
f
= p
2
x
+p
2
y
,
we have
v
f
=
_
p
2
x
+p
2
y
m
1
+m
2
=
_
1
(6 kg) + (18 kg)
_

_
(46436.3 kg
2
m
2
/s
2
)
+ (4946.53 kg
2
m
2
/s
2
)
_
1/2
= 9.44491 m/s .
007 10.0 points
Two weights attached to a uniform beam of
mass 52 kg are supported in a horizontal po-
sition by a pin and cable as shown in the
gure.
13 kg
42 kg
3 m
4.4 m
27

52 kg
What is the tension in the cable which sup-
ports the beam? The acceleration of gravity
is 9.8 m/s
2
.
1. 0.993261
2. 0.905703
3. 2.24204
4. 1.65921
5. 0.58395
6. 1.1684
7. 1.14228
8. 0.812861
9. 1.52455
10. 0.78371
Correct answer: 1.65921 kN.
Explanation:
Let : m = 52 kg ,
M
1
= 13 kg ,
M
2
= 42 kg ,
L
1
= 3 m,
L
2
= 4.4 m, and
= 27

.
M
1
M
2
L
1
L
2


F = 0 and

= 0 .
The sum of the torques about the pivot is
T L
2
sin mg
L
2
2
M
1
g L
1
M
2
g L
2
= 0
T =
mg
2 sin
+
M
1
g L
1
L
2
sin
+
M
2
g
sin
=
(52 kg)(9.8 m/s
2
)
2 sin 27

+
(13 kg) (9.8 m/s
2
) (3 m)
(4.4 m) sin27

+
(42 kg)(9.8 m/s
2
)
sin 27

= 1659.21 N = 1.65921 kN .
008 (part 1 of 2) 10.0 points
A small putty of putty of mass m falls from
the ceiling and lands on the outer rim of a
turntable of radius R and moment of inertia
I
0
that is rotating freely with angular speed

i
about its vertical xed symmetry axis.
Version 091 EX3 ditmire (58216) 7
What is the post-collision angular speed of
the turntable plus putty?
1.
f
=

i
2 +
mR
2
I
0
2.
f
=

i
1 +
mR
3
I
0
3.
f
=

i
1 +
mR
2
I
0
correct
4.
f
=

i
1 +
mR
I
0
Explanation:
The nal rotational inertia of the turntable-
plus-putty is
I
f
= I
0
+I
blob
= I
0
+mR
2
.
Using conservation of angular momentum,
I
0

i
= I
f

f

f
=
I
0

i
I
f

f
=
I
0

i
I
0
+mR
2
=

i
1 +
mR
2
I
0
.
009 (part 2 of 2) 10.0 points
After several turns, the putty ies o the edge
of the turntable.
What is the angular speed of the turntable
after the putty ies o?
1.

=

i
1 +
mR
2
I
0
correct
2.

=

1 +
mR
2
I
0
3.

=

i
1 +
mR
3
I
0
4.

=

i
1 +
mR
I
0
Explanation:
If the putty ies o tangentially to the
turntable, its angular momentum doesnt
change (with respect to an axis through the
center of turntable). Because there is no ex-
ternal torque acting on the putty-turntable
system, the total angular momentum of the
system will remain constant and the angular
momentum of the turntable will not change.
Because the moment of inertia of the table
hasnt changed either, the turntable will con-
tinue to spin at

=
f
.
010 (part 1 of 2) 10.0 points
A 3 kg bicycle wheel rotating at a
2336 rev/min angular velocity has its shaft
supported on one side, as shown in the gure.
When viewing from the left (from the positive
x-axes), one sees that the wheel is rotating in
a clockwise manner. The distance from the
center of the wheel to the pivot point is 0.5 m.
The wheel is a hoop of radius 0.6 m, and its
shaft is horizontal.
Assume all of the mass of the system is
located at the rim of the bicycle wheel. The
acceleration of gravity is 9.8 m/s
2
.
y
z
x
0
.
5
m
mg

2
3
3
6
r
e
v
/
m
i
n

0.6 m
radius
Find the change in the precession angle
after a 1.7 s time interval.
1. 6.97485
2. 2.59689
3. 4.50409
4. 6.87296
5. 5.41956
6. 5.74286
7. 8.96267
8. 11.5393
9. 12.8698
Version 091 EX3 ditmire (58216) 8
10. 5.03432
Correct answer: 5.41956

.
Explanation:
Let : m = 3 kg ,
= 2336 rev/min
=
2 (2336 rev/min)
(60 s/min)
= 244.625 rad/s ,
b = 0.5 m, and
R = 0.6 m.
Basic Concepts:
=
d

L
dt
Solution: The magnitude of the angular
momentum I of the wheel is
L = I = mR
2
,
and is along the negative x-axis.
From the gure in Part 2, we get =
L
L
.
Using the relation, L = t , where is the
torque, =

b mg .
The precession angle is
=
L
L
=
t
L
=
mg b t
mR
2

=
g b t
R
2

=
(9.8 m/s
2
) (0.5 m) (1.7 s)
(0.6 m)
2
(244.625 rad/s)
= 0.0945891 rad
= 5.41956

.
011 (part 2 of 2) 10.0 points
The direction of precession as viewed from the
top is
1. counter-clockwise.
2. clockwise. correct
3. opposite to the direction of rotation of the
wheel.
4. along the direction of rotation of the
wheel.
5. static, since angular momentum is con-
served.
Explanation:
=

b mg , where

b is along the positive
y-axis and g is into the page, in the gure
below. Therefore the direction of the torque
is along the positive y-axis.
+x x
L
+

wheel
Viewed from Above
+y
L
We can see the direction of precession is
clockwise, due to
=

L
t
,
that is L is in the direction of the torque
and is producing clockwise motion (

L
produces clockwise motion).
012 10.0 points
Two particles of masses m and 3 m are
moving toward each other along the x-axis
with the same speed v. They undergo a head-
on elastic collision and rebound along the x-
axis.
m
v
3 m
v
Determine the nal speed of the heavier
object.
Version 091 EX3 ditmire (58216) 9
1. v

3m
=
2
3
v
2. v

3m
=
3. v

3m
= v
4. v

3m
= 2 v
5. v

3m
=
1
2
v
6. v

3m
=
3
2
v
7. v

3m
=
1
3
v
8. v

3m
= 0 correct
9. v

3m
= 4 v
10. v

3m
= 3 v
Explanation:
Let : v
1
= v ,
v
2
= v ,
m
1
= m and
m
2
= 3 m.
Basic Concepts:
Conservation of momentum
Solution:
Denote the smaller mass initially moving to
the right as m
1
and the larger mass initially
moving to the left as m
2
. Since no external
forces act on the two masses, even during the
collision, the total momentum is conserved.
Also, since the masses collide elastically, en-
ergy is conserved. Since this is a head on,
elastic collision we can use
v

2
= 2 v
cm
v
2
, where
v
cm
=
m
1
v
1
+m
2
v
2
m
1
+m
2
, so
v

2
=
2 m
1
v
1
+ 2 m
2
v
2
m
1
+m
2

v
2
(m
1
+m
2
)
m
1
+m
2
=
2 mv 6 mv +mv + 3 mv
m+ 3 m
= 0 .
Alternative Solution: From the relative
velocities formula,
v
1
v
2
= v

2
v

1
. (1)
From conservation of momentum,
m
1
v
1
+m
2
v
2
= m
1
v

1
+m
2
v

2
. (2)
Combining Eqs. 1 and 2 gives
m
1
v
1
+m
2
v
2
= m
1
(v

2
+v
2
v
1
) +m
2
v

2
,
v

2
=
m
1
v
1
+m
2
v
2
m
1
(v
2
v
1
)
m
1
+m
2
=
mv + 3 m(v) m(v v)
m+ 3 m
=
m(v 3 v +v +v)
4 m
= 0 .
013 10.0 points
An Atwood machine is constructed using a
hoop with spokes of negligible mass. The
2.1 kg mass of the pulley is concentrated on
its rim, which is a distance 21.1 cm from the
axle. The mass on the right is 1.01 kg and on
the left is 1.61 kg.
3.2 m
2.1 kg
21.1 cm

1.61 kg
1.01 kg
What is the magnitude of the linear acceler-
ation of the hanging masses? The acceleration
of gravity is 9.8 m/s
2
.
1. 1.24576
2. 1.5377
3. 1.03715
4. 1.4
Version 091 EX3 ditmire (58216) 10
5. 0.759279
6. 1.09776
7. 0.655906
8. 1.17297
9. 0.709589
10. 0.735
Correct answer: 1.24576 m/s
2
.
Explanation:
Let : M = 2.1 kg ,
R = 21.1 cm,
m
1
= 1.01 kg ,
m
2
= 1.61 kg ,
h = 3.2 m, and
v = R.
Consider the free body diagrams
1.61 kg 1.01 kg
T
2
T
1
m
2
g
m
1
g
a a
The net acceleration a = r is in the direc-
tion of the heavier mass m
2
.
For the mass m
1
,
F
net
= m
1
a = m
1
g T
1
T
1
= m
1
g m
1
a
and for the mass m
2
,
F
net
= m
2
a = T
2
m
2
g
T
2
= m
2
a +m
2
g .
The pulleys mass is concentrated on the
rim, so I = M r
2
, and

net
=


ccw


cw
= I
T
1
r T
2
r = (mr
2
)
_
a
r
_
= mr a
ma = T
1
T
2
ma = (m
1
m
2
) g (m
1
+m
2
) a
ma + (m
1
+m
2
) a = (m
1
m
2
) g
a =
(m
1
m
2
) g
m+m
1
+m
2
=
(1.01 kg 1.61 kg) (9.8 m/s
2
)
2.1 kg + 1.01 kg + 1.61 kg
= 1.24576 m/s
2
.
014 10.0 points
A machine part rotates at an angular speed
of 0.37 rad/s; its speed is then increased to
1.46 rad/s using an angular acceleration of
0.41 rad/s
2
.
Find the angle through which the part ro-
tates before reaching this nal speed.
1. 10.5369
2. 0.790455
3. 1.47401
4. 1.372
5. 2.54803
6. 3.135
7. 4.74337
8. 5.13939
9. 18.96
10. 2.43256
Correct answer: 2.43256 rad.
Explanation:
Let :
0
= 0.37 rad/s ,
= 1.46 rad/s , and
= 0.41 rad/s
2
.
From kinematics

2
=
2
0
+ 2
=

2

2
0
2
=
(1.46 rad/s)
2
(0.37 rad/s)
2
2 (0.41 rad/s
2
)
= 2.43256 rad .
Version 091 EX3 ditmire (58216) 11
015 10.0 points
Pat builds a track for his model car out of
wood. The track is 4 cm wide (along the z
coordinate), 1 m high (along the y coordi-
nate) and 2.5 m long (along the x coordinate
starting from x = 0).
The runway is cut such that it forms part
of the left-hand side of a parabola, see gure
below.
x
1
m
y
2.5 m
y =
1
(6.25 m)
[x (2.5 m)]
2
4 cm
Locate the horizontal position of the center
of gravity of this track.
1. 0.625
2. 0.75
3. 0.675
4. 0.775
5. 0.825
6. 0.725
7. 0.7
8. 0.6
9. 0.85
10. 0.65
Correct answer: 0.625 m.
Explanation:
Let : a = 2.5 m,
b = a
2
= 6.25 m,
z = 4 cm, and
= surface density .
Basic Concepts:
r
CM
=
_
r dV
_
dV
=
_
r dV
M
x
1
m
y
2.5 m
y =
1
(6.25 m)
[x (2.5 m)]
2
4 cm
x
dx
Solution: Let represent the mass-per-
face area. A vertical strip at position x, with
width dx and height
(x a)
2
b
, has mass
dm =
(x a)
2
b
dx .
The total mass is
M =
_
dm =
_
2.5 m
x=0

(x a)
2
b
dx
=

b
_
2.5 m
0 m
(x
2
2 a x +b) dx
=

b
_
x
3
3
a x
2
+b x
_

2.5 m
0 m
.
The x coordinate of the center of gravity is
x
cg
=
1
M
_
x dm
=
1
M
_
2.5 m
x=0
x (x a)
2
dx
b
=

b M
_
2.5 m
x=0
x (x a)
2
dx
=

b M
_
2.5 m
0 m
(x
3
2 a x
2
+b x) dx
=

b M
_
x
4
4

2 a x
3
3
+
b x
2
2
_

2.5 m
0 m
=
x
4
4

2 a x
3
3
+
b x
2
2
x
3
3
a x
2
+b x

2.5 m
0 m
Version 091 EX3 ditmire (58216) 12
=
3.25521 m
2
5.20833 m
= 0.625 m.
016 10.0 points
A student of mass 54 kg wants to walk
beyond the edge of a cli on a heavy beam of
mass 250 kg and length 5.5 m. The beam is
not attached to the cli in any way, it simply
lies on the horizontal surface of the clitop,
with one end sticking out beyond the clis
edge:
d
How far from the edge of the ledge can the
beam extend if it sticks out as far as possible
beyond the edge, but the student can walk to
the beams end without falling down?
1. 4.46581
2. 2.83562
3. 2.28784
4. 5.074
5. 3.69955
6. 3.01326
7. 2.56881
8. 4.68992
9. 2.26151
10. 3.33333
Correct answer: 2.26151 m.
Explanation:
Let : m = 54 kg ,
M = 250 kg , and
L = 5.5 m.
The stability of the beam (with the student
standing on it) requires that the center of
gravity of the beam + student system must
lie above the cli itself. If this overall center of
gravity were to move beyond the clis edge,
the beam would tilt down and fall o the cli.
Let the clis edge be the origin of our co-
ordinate system. The beam extends from
X
1
= d L < 0 (on the cli) to X
2
= d > 0
(o the cli), so assuming the beam is uni-
form, its center of mass is located at
X
beam
CM
=
X
1
+X
2
2
= d
L
2
.
When the student walks all the way to the
beams end, his own center of mass is located
at
X
student
CM
d ,
where the approximation neglects the size
of the student compared to the beams
length. The overall center of mass of the
beam + student system is therefore located at
X
overall
CM
=
m
m+M
X
student
CM
+
M
m+M
X
beam
CM
=
md
m+M
+
M
m+M
_
d
L
2
_
= d
M
m+M
L
2
< 0
for overall stability, so
d < d
max
=
M
m+M
L
2
=
250 kg
54 kg + 250 kg
5.5 m
2
= 2.26151 m .
017 (part 1 of 2) 10.0 points
Two rods of uniform density, length and
mass m, are shown. Rod B is pivoted about
a point
1
3
in from the left end and the other
A is pivoted about its left end. They are held
horizontally and then released.
A

Version 091 EX3 ditmire (58216) 13


Which expression best describes the rela-
tionship between the torque felt by rod A
(pivoted about its end) and rod B, (pivoted
about a point
1
3
down from its end)?
1.
A
=
B
2. Not enough information is given.
3.
A
<
B
4.
A
>
B
correct
Explanation:
The center of mass is

2
from the pivot on
A, so

A
= mg
_

2
_
.
Gravity acts at

2
but the distance from the
focal point is

6
from B, so

B
= mg
_

6
_
.
Therefore, rod A experiences the greater
torque.
018 (part 2 of 2) 10.0 points
Which expression best describes the relation-
ship between the angular acceleration of rod
A versus that of rod B?
1.
A
>
B
2.
A
<
B
3. There is not enough information.
4.
A
=
B
correct
Explanation:
From rotational kinematics
= I ,
the moment of inertia for rod A is
I
A
=
1
3
m
2
and for rod B
I
B
=
1
12
m
2
+m
_

6
_
2
=
1
9
m
2
,
For rod A

A
=

A
I
A
=
mg
2
1
3
m
2
=
3 g
2
and for rod B

B
=

B
I
B
=
mg
6
1
9
m
2
=
3 g
2
.
Therefore

A
=
B
.
keywords:
019 10.0 points
A ladder is leaning against a smooth wall.
Assume there is friction between the ladder
and the horizontal oor. Adjust the ladder
until the critical situation is reached; i.e., it is
at the border line between the stable and the
unstable cases.
A weight comparable to the weight of the
ladder is carefully hung at the center of mass
of the ladder.
mg
L
f

h
b
F
N

What will happen to the ladder-weight sys-


tem?
Version 091 EX3 ditmire (58216) 14
1. It becomes unstable. A new critical situ-
ation may be reached by increasing .
2. It becomes more stable. A new critical
situation may be reached by increasing .
3. It is still at the critical situation. correct
4. It becomes more stable. A new critical
situation may be reached by decreasing .
5. It becomes unstable. A new critical situ-
ation may be reached by decreasing .
Explanation:
The critical angle is independent of the
mass of the ladder, which implies that it is
independent of any additional mass placed at
the center of mass of the ladder.
020 10.0 points
A 1.5 kg bicycle wheel with a radius of 0.27 m
turns at a constant angular speed of 22 rad/s
when a(n) 0.31 kg reector is at a distance of
0.13 m from the axle.
What is the angular speed of the wheel
when the reector slides to a distance of 0.20
m from the axle?
1. 20.9438
2. 29.801
3. 24.4978
4. 22.9303
5. 20.706
6. 26.1084
7. 19.9138
8. 21.6812
9. 26.9722
10. 27.665
Correct answer: 20.706 rad/s.
Explanation:
Basic Concepts:
The bicycle wheel can be modeled as a thin
hoop, and the reector as a point mass.
L
wheel
= I
wheel
= MR
2

L
refl
= I
refl
= mr
2

L
f
= L
i
Given:
M = 1.5 kg
R = 0.27 m

i
= 22 rad/s
m = 0.31 kg
r
i
= 0.13 m
r
f
= 0.20 m
Solution: Angular momentum is conserved,
so
L
wheel,f
+L
refl,f
= L
wheel,i
+L
refl,i
I
wheel

f
+I
refl,f

f
= I
wheel

i
+I
refl,i

i
_
MR
2
+mr
2
f
_

f
=
_
MR
2
+mr
2
i
_

f
=
MR
2
+mr
2
i
MR
2
+mr
2
f

i
=
(1.5kg)(0.27m)
2
+(0.31kg)(0.13m)
2
(1.5kg)(0.27m)
2
+(0.31kg)(0.2m)
2
(22 rad/s)
= 20.706 rad/s

Vous aimerez peut-être aussi